whats 10 + 4? i need to know if you answer you get 25 points

Answers

Answer 1
the answer is 14, 0+4 is 4 and then you put the 1 infront of the 4 making 14
Answer 2
The answer would be 14.

Related Questions

Help me out!!!!!!!!!!!!!

Answers

Answer:

Add all of the sides up:

x^2 + y + 6x + 2y + y^2 + 6x + 2y + y^2 + 4x^2

= 5x^2 + 12x + 2y^2 + 5y


The question is:
Jason has 20 gallons of a 12% alcohol solution. How many gallons, to the nearest tenth, must be replaced by a 36% alcohol solution to give 20% gallons of 15% alcohol solution.

I need and explanation on how to set it up and do it. I have a test today and I don’t understand it.

Answers

Answer:

2.5 gallons of the 20% solution must be replaced by the 36% solution.

Step-by-step explanation:

[tex].12(20 - x) + .36x = 20(.15)[/tex]

[tex]2.4 - .12x + .36x = 3[/tex]

[tex]2.4 - .24x = 3[/tex]

[tex] - .24x = - .6[/tex]

[tex].24x = .6[/tex]

[tex]x = 2.5[/tex]

The zeros of a parabola are -4 and 2 and (6,10) is a point on the graph which equation can be solved to determine the value of a in the equation of the parabola

Answers

Answer:

Let consider the following linear equation systems by using the known points and second-grade polynomial:

(-1, 7)

a + b + c = 7a+b+c=7

(5, 7)

25\cdot a + 5\cdot b + c = 725⋅a+5⋅b+c=7

(6,10)

36\cdot a + 6 \cdot b + c = 1036⋅a+6⋅b+c=10

After some algebraic manipulation, the values for the polynomial coefficients are found:

a = \frac{3}{5}a= 53 , b = -\frac{18}{5}b=− 518

, c = 10c=10

The polynomial is:

y = \frac{3}{5}\cdot x^{2} - \frac{18}{5}\cdot x+10y= 53

Step-by-step explanation:

For complete solution see the above attachment!

Find the point Q along the directed line segement from point R (-3, 3) to point S(6, -3) that divides the segment in the ratio 2:1.

A. (3, 0)
B. (6, -4)
C. (0, 1)
D. (3, -1)

Answers

Answer: (3,-1)

Step-by-step explanation:

distance between R & S x values: 9

distance between R & S y values: -6

multiply each by 2/3 (the proportion)

9/1 x 2/3 = 6; -6/1 x 2/3= -4

add the answers to R x & y values:

-3 + 6= 3; 3 - 4= -1

Answer:

D

Step-by-step explanation:

using the section formula

(x₁, y₁ ) and (x₂, y₂ ) divided in the ratio m : n , then

point = ([tex]\frac{mx_{2}+nx_{1} }{m+n}[/tex] , [tex]\frac{my_{2}+ny_{1} }{m+n}[/tex] )

thus for R (- 3, 3 ) and S (6, - 3 ) in the ratio 2 : 1

Q = ( [tex]\frac{2(6)+1(-3)}{2+1}[/tex] , [tex]\frac{2(-3)+1(3)}{2+1}[/tex] )

   = ( [tex]\frac{12-3}{3}[/tex] , [tex]\frac{-6+3}{3}[/tex] )

  = ( [tex]\frac{9}{3}[/tex] , [tex]\frac{-3}{3}[/tex] )

  = (3, - 1 )

pleasee help me with this function asap

Answers

Answer:

  b, c

Step-by-step explanation:

A function is continuous if its graph can be drawn without lifting the pencil. It is decreasing wherever its slope is negative.

__

A graph of the function is attached. It has a "jump" discontinuity at x=0, so is not a continuous function.

The value of f(0) is 2, so the y-intercept is 2.

The given function is defined for all values of x, so its domain is all real numbers.

The function is decreasing for values of x > 0, so does not approach positive infinity for large positive x.

The function has a stationary point at x=0, so is not decreasing over its entire domain.

_____

Additional comment

The function is decreasing everywhere except at x=0. The point (0, 2) is the vertex of the quadratic portion of the function, so a tangent is horizontal there. At such horizontal tangent points, a function is neither increasing nor decreasing. It is tempting to ignore this exception, because the function is decreasing everywhere else.

The average of three unique number is 50.if the loweat number is 30 what is the sum of other two numbers

Answers

Answer:

The sum of the other two numbers is 120.

Step-by-step explanation:

To find an average, you add all the numbers involved and divide it by the amount of numbers involved. in this case,

(x+y+z) ÷ 3 =50

x=30

Divide both sides by three to remove the denominator.

(30+y+z)÷ 3 (*3) = 50(*3)

(30+y+z)=150

Subtract 30 from both sides.

(30+y+z)-30= 150-30

y+z=120

The table represents the function f(x).

A 2-column table with 7 rows. The first column is labeled x with entries negative 4, negative 3, negative 2, negative 1, 0, 1, 2. The second column is labeled f of x with entries negative 66, negative 29, negative 10, negative 3, negative 1, 6.

When f(x) = –3, what is x?

Answers

Answer:

-1/2

Step-by-step explanation:

Answer:x is -1

Step-by-step explanation:

Find the area of the smaller sector.
Round to the nearest tenth.
50°
7.13 ft
Area = [?]ft²

Answers

Answer:

A ≈ 22.2 ft²

Step-by-step explanation:

the area (A) of the sector is calculated as

A = area of circle × fraction of circle

   = πr² × [tex]\frac{50}{360}[/tex]

   = π × 7.13² × [tex]\frac{5}{36}[/tex]

    = 50.8369π × [tex]\frac{5}{36}[/tex]

    ≈ 22.2 ft² ( to the nearest tenth )

Answer: 22.2 ft^2

Step-by-Step Solution:

Radius (r) = 7.13 ft
Angle (θ) = 50°

Area of a Sector = θ/360 * πr^2

Therefore,
= θ/360 * πr^2
= θ/360 * π * r * r
= 50/360 * 3.14 * 7.13 * 7.13
= 50/360 * 3.14 * 50.8369
= 5/36 * 159.63
= 798.15/36
= 22.17
=> 22.2

Area of the Sector = 22.2 ft^2


Triangle 304 degrees and 115 degrees work out x

Answers

a straight line make an angle of 180° ,

so 115+y=180

y=65°

for the top part of the triangle it makes an angle of 360° , so 304+y=360

y=56

sum measure of interior angles of triangle is 180° , According to this rule (n-2)180 ; where n is the no. of sides .

so

65+56+y= 180

121+y=180

y=59( the angle next to X)

X+59=180( together they make an angle of 180)

x=121°

Hope it helps

please give brainliest

Find the sum of the largest and the smallest of the following four numbers:
3.09 3.9 0.39 0.399


PLEASE HELP

Answers

Answer:

Using the concepts of Number System, we find that the sum of the largest and the smallest of the given four numbers is 4.29.

Step-by-step explanation:

The given set of numbers is 3.09, 3.9, 0.39 and 0.399.

The given numbers are related to each other as:

3.9 > 3.09 > 0.399 > 0.39

3.9 is greater than 3.09 as

3.9 > 3.09

3 + 0.9 > 3 + 0.09

0.9 > 0.09

90 > 9

0.399 is greater than 0.39 as

0.399 > 0.39

399 > 390

Now, the smallest number in the series is 0.39 and the largest number in the series is 3.9.

The sum of the largest and the smallest of the given four numbers is:

3.9 + 0.39

4.29

For more explanation, refer the following link:

https://brainly.com/question/9842110

#SPJ10

HELP
Can someone help me with number 26 plss

Answers

Answer:

(-2,-9)

(-1,8)

(0,-5)

(1,0)

(2,7)

Step-by-step explanation:

Solve equation by using the quadratic formula. 6x^2 +2x =4?
A) x= 3/2, 1 B) x= 2/3, -1 C) x=3/2, -1 D) x=3/2, 0

Answers

Answer:

Answer x=2/3, -1
Option B

Step-by-step explanation:

Answer:

B) x =( 2/3, -1)

Step-by-step explanation:

Equation:

[tex]6x {}^{2} + 2x = 4[/tex]

Solution:

We know that,

[tex] \rm \: Quadratic \: formula (x)=\cfrac{-b\pm\sqrt{b^2-4ac}}{2a}[/tex]

According to the problem,

a = 6b = 2c = -4

Plugging them on the formulae,we obtain,

[tex]x = \cfrac{ - 2 \pm \sqrt{ 2 {}^{2} - 4 \{6 \times ( - 4) \} } }{2 \times 6} [/tex]

[tex]x = \cfrac{ - 2 \pm \: \sqrt{4 - 4 \{ - 24 \} } }{12} [/tex]

[tex]x = \cfrac{ - 2 \pm \: 10 }{12} [/tex]

[tex]x = \cfrac{ - 1 \pm5}{6} [/tex]

[tex]\boxed{x = \cfrac{2}{3} \: \rm or - 1}[/tex]

Choice B is accurate.

The line ax − by + 3 = 0 is parallel to the line 3x + 2y − 4 = 0 and passes through the point (1, −2). Find a and b.

Answers

Answer:

a=9

b=-6

Step-by-step explanation:

If the nth term of a sequence is 18-8n what is the 8th term

Answers

Answer:

[tex]a_{n} =18-8n\\a_{8} =18-8(8)=-46[/tex]

Answer:

-46

Step-by-step explanation:

(hello, I just answered your previous question--some stuff will sound similar because we are applying the same concept :)  )

We know that the nth term of the sequence is 18 - 8n

the "nth" term of a sequence means that you can plug in the term place to get the term value (ex. if the nth term is n + 2, we can plug any term in {let's say, the eighth term, 8 + 2 = 10 ; to get our term's value}  )

  18 - 8n is sequence

  18 - 8(8) is 8th term

  18 - 64 is 8th term

 -46 is the 8th term

hope this helps!! :)

(06.06)

Which of the following demonstrates the Commutative Property of Multiplication?

5(2a − 3) = 10a − 15
10a − 15 = (2a − 3) ⋅ 5
5(2a − 3) = (2a − 3) ⋅ 5
(5 ⋅ 2a) − 3 = 5(2a − 3)

Answers

The correct answer is option C which is 5(2a − 3) = (2a − 3) ⋅ 5 showing commutative property.

What is commutative property?

If altering the operands' order has no effect on the outcome, the binary operation is commutative in mathematics.

a x b = b x a

So for the expression 5(2a − 3) = (2a − 3) ⋅ 5 suppose 5 is a and ( 2a - 3 ) is b so.

a x b = b x a

5(2a − 3) = (2a − 3) ⋅ 5

Therefore the correct answer is option C which is 5(2a − 3) = (2a − 3) ⋅ 5 showing commutative property.

To know more about commutative property follow

https://brainly.com/question/2475734

#SPJ1

need help asap! Fill in the blank by entering just a number…

Answers

Answer:

x = 6

Step-by-step explanation:

CT is a median, which means it divided AB into two equal parts so we can write the following equation:

7x + 9 = 5x + 21 transfer like terms to the same side of the equation

7x - 5x = 21 - 9 do the extraction

2x = 12 divide both sides by 2

x = 6

There are 12 ducks and 11 geese in a certain park. Find the ratio of geese to ducks.

Answers

The ratio of the geeses to ducks will be 11:12 because there are 11 geeses and 12 ducks :)

como identifico una X Cubica en un polinomio

Answers

la componente "cubica" será aquella que tenga un exponente igual a 3.

¿como identifico una X Cubica en un polinomio?

Un polinomio tiene la forma general:

[tex]p(x) = a_n*x^n + ... + a_1*x^1 + a_0[/tex]

Donde todos los exponentes de x son numeros naturales.

Particularmente, la componente "cubica" será aquella que tenga un exponente igual a 3.

Entonces, por ejemplo, en:

[tex]p(x) = 4*x^5 + 2*x^4 + 5*x^3[/tex]

La parte cubica será:

[tex]5*x^3[/tex]

Sí quieres aprender más sobre polinomios:

https://brainly.com/question/4142886

#SPJ1

Given the following formula,A school needs to buy new notebook and desktop computers for its computer lab. The notebook computers cost $300 each, and the desktop computers cost $225 each. How much would it cost to buy 18 notebooks and 12 desktop computers? How much would it cost to buy n notebooks and dd desktop computers?

Total cost, 18 notebooks and 12 desktop computers:

Total cost, n notebooks and dd desktop computers:

Answers

The cost to buy 18 notebooks and 12 desktop computers will be $8100.

The cost to buy n notebooks and d desktop computers will be 300n + 225d

What is the cost of a given material?

The cost of a given material is the amount attached and to be paid for getting that material.

From the given information:

The cost of notebook computers = $300The cost of desktop computers is $225 each

The cost to buy 18 notebooks and 12 desktop computers will be:

= 300(18) + 225(12)

= 5400 + 2700

= $8100

The cost to buy n notebooks and d desktop computers will be:

= 300(n) + 225(d)

= 300n + 225d

Learn more about determining the cost of material here:

https://brainly.com/question/21720717

#SPJ1

Which of the following values can be found by using the formula √1+tan^2 0?

Answers

√1+tan²Ø√1+sin²Ø/cos²Ø√cos²Ø+sin²Ø/cos²Ø√1/cos²Ø1/cosØsecØ

cos and sec can be found

The requried simplified expression is 1 / cosØ, which is equivalent to secØ. The correct option is secØ.

Let's simplify the given expression and see which of the provided options match:

We are given the expression √(1 + tan²Ø).

Now, recall the trigonometric identity: tan²Ø = sin²Ø / cos²Ø.

Substitute this identity into the expression:

√(1 + sin²Ø / cos²Ø)

Next, find a common denominator for the fraction inside the square root:

√((cos²Ø + sin²Ø) / cos²Ø)

Since cos²Ø + sin²Ø = 1 (as per the Pythagorean trigonometric identity), the expression becomes:

√(1 / cos²Ø)

Now, we can take the square root of 1 and cos²Ø separately:

√1 / √cos²Ø

Simplify:

1 / cosØ

The simplified expression is 1 / cosØ, which is equivalent to secØ.

Therefore, the correct option is: secØ.

Learn more about trigonometry here:

https://brainly.com/question/20885025

#SPJ4

How tall is the tree?
5ft
35°
-20ft-
[? ]ft

Answers

Answer:

[tex]\boxed{\bf 19ft}[/tex]

Step-by-step explanation:

[tex]\bf tan\:\theta =\cfrac{opp}{adj}[/tex]

[tex]\bf tan(35^o)=\cfrac{x}{20}[/tex]

[tex]\bf 20\;tan (35^o)=x[/tex]

[tex]\bf 14ft=x[/tex]

The height of the tree:-

[tex]\bf x+5ft[/tex][tex]\bf 14ft+5ft[/tex][tex]\bf 19ft[/tex]

Answer:

19.00 ft to the nearest hundredth.

Step-by-step explanation:

tan 35 = h / 20

h = 20 tan 35

Now we add the man's height

Total height of the tree

=  20 tan 35 + 5

=  19.00 ft to the nearest hundredth.

Complete the tasks to subtract the polynomials vertically. (1.3t3 0.4t2 – 24t) – (0.6t2 8 – 18t) what is the additive inverse of the polynomial being subtracted?

Answers

The difference of the polynomials is [tex]1.3t^{3}-0.2t^{2}-6t-8[/tex] and the additive inverse of the polynomial being subtracted is [tex]-0.6t^{2} +8+18t[/tex].

The given polynomials are [tex](1.3t^{3}+0.4t^{2} -24t )[/tex] and [tex]0.6t^{2} +8-18t[/tex].

We need to find the additive inverse of the polynomial being subtracted.

What is additive inverse?

In mathematics, the additive inverse of a number a is the number that, when added to a yields zero. This number is also known as the opposite, sign change, and negation.

Now, [tex](1.3t^{3}+0.4t^{2} -24t )-(0.6t^{2} +8-18t)[/tex]

[tex]=1.3t^{3}+0.4t^{2}-0.6t^{2}-24t+18t-8[/tex]

[tex]=1.3t^{3}-0.2t^{2}-6t-8[/tex]

The additive inverse of the polynomial being subtracted is [tex]-0.6t^{2} +8+18t[/tex].

Therefore, the difference of the polynomials is [tex]1.3t^{3}-0.2t^{2}-6t-8[/tex] and the additive inverse of the polynomial being subtracted is [tex]-0.6t^{2} +8+18t[/tex].

To learn more about the additive inverse visit:

https://brainly.com/question/13715269.

#SPJ1

Kit and Kat are building a kite for the big kite festival. Kit has already
cut his sticks for the diagonals. He wants to position P so that he will
have maximum kite area. He asks Kat for advice. What should Kat
tell him?

Answers

Kat should tell Kim that It's too late to change the area of this kite because the length of the diagonals of a kite determines its area.

How to calculate the area of a kite?

In Mathematics, the area of a kite is equal to one-half the product of the length of its diagonals. Mathematically, the area of a kite can be calculated by using this formula:

A = ½ × d₁ × d₂

Where:

A is the area of a kite.d₁ and d₂ are the length of the diagonals of a kite.

Since Kit has already cut his sticks for the diagonals, Kat should tell him that it's too late to change the area of this kite because the length of the diagonals of a kite determines its area.

Read more on area of a kite here: https://brainly.com/question/9593985

#SPJ1

Fatoumata just accepted a job at a new company where she will make an annual salary of $43000. Fatoumata was told that for each year she stays with the company, she will be given a salary raise of $5000. How much would Fatoumata make as a salary after 3 years working for the company? What would be her salary after tt years?

Answers

Fatoumata's salary after 3 years will be $58,000.

Fatoumata's salary after tt years will be $43,000 + (5,000tt).

What is Fatoumata's salary?

The linear equation that can be used to determine Fatoumata's salary after the passage of time is:

Initial salary + (yearly increase x number of years)

$43,000 + 5000t

$43,000 + 5000(3) =  $58,000.

To learn more about linear equations, please check: https://brainly.com/question/26434260

#SPJ1

write the equation of the line that contains (-4, -3) and is parallel to the line x+2y=-10

Answers

Introduction

Hello! I hope you are having a nice day. My name is Galaxy and I will be helping you with this problem. We can solve this problem in 2 steps, respectively Theory and Solving.

I'll go ahead and start with the Theory.

Theory

Before we attempt to solve the problem mathematically, we must first figure out how we're going to solve this problem.  

We know that we have a line and a point, we can start by graphing the equation and point that we've received.

Solving

Now that we have our points plotted and our equations graphed, we can start to see that something odd is happening, the given point is on the line itself.

We can check this by inputting the points into our equation:

[tex](-4)+2(-3)=-10?\\(-4)+(-6)=-10?\\-10=-10[/tex]

This proves that our point is on the line that we were given.

According to the basic rules of Euclidean Geometry, we know that parallel lines can never intersect each other and due to this there cannot be any solution to this problem.

Any other line using this point would intersect the given line, and due to this, this problem has no real solutions.

* The only line that can use these points and graph is the line provided, and that cannot work due to the lines intersecting at an infinite number of points.

Cheers,
Galaxy.

What is m∠C?

Enter your answer in the box.

Answers

Answer:

60

Step-by-step explanation:

Total degree of triangle interior angles is 180

So x+2x+3x=180

6x=180

x=30

Measurement of angle C is 2x which is 60 degree

Explain how you can prove the difference of two cubes identity. a3 – b3 = (a – b)(a2 ab b2)

Answers

Answer:

Step-by-step explanation:

Expanding the right-hand side,

[tex](a-b)(a^{2}+ab+b^{2})=a^{3}+a^{2}b+ab^{2}-a^{2}b-ab^{2}-b^{3}=a^{3}-b^{3}[/tex]

Thus, the identity is true.

#+3)(2+1) for x = 3.
c+5)(x-5)

Answers

Answer:

x=1/3

Step-by-step explanation:

first multiply both bracket to together and then you get, 9x=3

x=1/3, then sub in to other equation and get the rest answer.

hope it helpful

This is confusing please help

Answers

Step-by-step explanation:

since we are using Pythagoras

c² = a² + b²

with c being the Hypotenuse (the baseline opposite of the 90° angle),

you want to get the expression

y² = 3² + 3²

so, you pick the ² and put it next to the y left of the "=" sign.

and to the right of the "=" sign you put 3, ², +, 3, ²

divided it by 7 and added
divided it by 7 and added 25. the result was 34 what was my number?
25. the result was 34 what was my number?

Answers

Answer:

your questions is not clear but from my observations

let the number be x

x/7+25=34

x+175/7=34

x+175=238

x=238-175

×=63

please mark me as brainlyst

Other Questions
A periodic wave has a wavelength of 0,50 m and a speed of 20 m/s, What is the wave frequency? What are noises made by very early humans called? oa pre-languages o b. proto-languages o c. premiere languages od. performance languages When the events of the story are told in the order in which they happen, the story is told What are the area in home economics that need the application of the element and the principles of design What is the slope of the line represented by the equation f(x) = -472x 4-4 PLS HELPPFill in each blank with the correct form of "ir" (to go). Use the verb chart on your vocab list to help!I to the cinema with my friends.Where you after school?Constable to the supermarket with his mom.We to the beach in July.They to church on Sundays.Maria and Carmen to my house at eight in the morning.She to the library to study.My friends and I to the park a lot.Students to the store to buy bookmarks.Where You? Name one Latino enclave in the United States Please help me answer question 7/11 asap While hiking through the Ural Mountains, Asia decided to use a walking stick to facilitate her climb.The best definition of the word that is underlined in the sentence above is __________. The tree diagram represents an experiment consisting of two trials. (Enter the probability to the hundredths place. Do not round.).6, A, .3, C, .7, D,.4, B, .2, C, .8, DP(D) = [ ? ]please no links Hawaii, alaska, maine, and vermont are the only four states to ban what?. Total quality management nike nmentsleesborationsryTeams (Web)nloadDown Browserent SuccesscerWhat type of aneurysm is pictured in the image provided?SaccularFusiformAorticO Dissecting Find the solution to the systemof equations.432GK3-4-3-2-1 1 2 3 4-1==x+2214234([?], [ ])Enter What does how are the readings situated within the historical, geopolitical context in which it's written/set mean? Read the passage. In it, the mother misuses the English idiom It takes two to tango. What does her language show about her as a character? She has picked up expressions and uses them like a native English speaker. She is at ease in her new home. She is embarrassed to speak. She has trouble communicating clearly in English. Graph the inequality.-6 y + 2x < 15 What were the historical precedents for the environmental activism of the 1970s?PLEASE HELP AND EXPLAIN WELL PLEASEEEE!! A machinist who had been producing 40 parts per day increased to the out put of 60 parts per day by going to a faster machine. How many times faster is the new machine? Wright your answer as a mixed number. Please help ASAPIs there any evidence that a reaction has occurred? (One or more answers may be true.)a. No, there is no change in the solution.b. Yes, because H2SO4 is being added.c. Yes, a precipitate formed.d. Yes, there is a change in conductivity.